试题

http://www.zizzs.com/gaokaomoni/

http://gaokao.xdf.cn/list_1023_1.html

\begin{Example}
(2011北京)椭圆$\displaystyle G: \frac{x^2}{4}+y^2=1$.过点$(m,0)$作图$x^2+y^2=1$的切线$l$交椭圆$G$于$A,B$两点.
\begin{enumerate}
\item[(I)] 求椭圆$G$的焦点坐标和离心率;

\item[(II)] 将$|AB|$表示为$m$的函数,并求$|AB|$的最大值.
\end{enumerate}
\end{Example}

\begin{Example}
(2011北京)若数列$A_n:a_1,a_2,\cdots,a_n(n\geq 2)$满足$|a_{k+1}-a_k|=1(k=1,2,\cdots,n-1)$,则称$A_n$为$E$数列.记$S(A_n)=a_1+a_2+\cdots+a_n$.
\begin{enumerate}
\item[(I)] 写出一个满足$a_1=a_5=0$,且$S(A_5)>0$的$E$数列$A_5$;

\item[(II)] 若$a_1=12,n=2000$,证明: $E$数列$A_n$是递增数列的充要条件是$a_n=2011$;

\item[(III)] 对任意给定的整数$n\, (n\geq 2)$,是否存在首项为$0$的$E$数列$A_n$,使得$S(A_n)=0$ ?如果存在,写出一个满足条件的$E$数列$A$;如果不存在,说明理由.
\end{enumerate}
\end{Example}
\begin{Proof}
\begin{enumerate}
\item[(I)] $0,1,2,1,0$是一个满足条件的$E$数列$A_5$.

(答案不唯一, $0,1,0,1,0;0,- 1,0,1,0$也是满足条件的$E$数列$A_5$)

\item[(II)] 必要性:因为$E$数列$A_n$是递增数列,所以$a_{k+1}-a_k=1\, (k=1,2,\cdots,1999)$,所以$A_n$是首项为$12$,公差为$1$的等差数列,所以$a_{2000}=12+(2000-1)\times 1=2011$.

充分性:由于$a_{2000}-a_{1999}\leq 1,a_{1999}-a_{1998}\leq 1,\cdots,a_2-a_1\leq 1$,所以$a_{2000}-a_1\leq 1999$,即$a_{2000}\leq a_1+ 1999$.又因为$a_1=12,a_{2000}=2011$,所以$a_{2000}=a_1+1999$.故$a_{k+1}-a_k=1>0\, (k=1,2,\cdots,1999)$,即$A_n$是递增数列.

综上,结论得证.

\item[(III)] 令$c_k=a_{k+1}-a_k=1>0\, (k=1,2,\cdots,n-1)$,则$c_k=\pm 1$.因为$a_2=a_1+c_1,a_3=a_1+c_1+c_2,\cdots,a_n=a_1+c_1+c_2+\cdots+c_{n-1}$,所以
\begin{align*}
S_n =& na_1+(n-1)c_1+(n-2)c_2+(n-3)c_3+\cdots+c_{n-1}\\
=&(n-1)+(n-2)+\cdots+1\\
&-[(1-c_1)(n-1)+(1-c_2)(n-2)+\cdots+(1-c_{n-1})]\\
=&\frac{n(n-1)}{2}-[(1-c_1)(n-1)+(1-c_2)(n-2)+\cdots+(1-c_{n-1})].
\end{align*}
因为$c_k=\pm 1$,所以$1-c_k$为偶数($k=1,2,\cdots,n-1$),所以$(1-c_1)(n-1)+(1-c_2)(n-2)+\cdots+(1-c_{n-1})$为偶数.

所以要使$S(A_n)=0$,必须使$\frac{n(n-1)}{2}$为偶数,即$4$整除$n(n-1)$,亦即$n=4m$或$n=4m+1$ ($m\in\mathbb{N}^\ast$).

当$n=4m$ ($m\in\mathbb{N}^\ast$)时, $E$数列$A_n$的项满足$a_{4k+1}=a_{4k-1}=0,a_{4k-2}=-1,a_{4k}=1$\, $(k=1,2,\cdots,m)$时,有$a_1=0,S(A_n)=0$;

当$n=4m+1$ ($m\in\mathbb{N}^\ast$)时, $E$数列$A_n$的项满足$a_{4k-1}=a_{4k-3}=0,a_{4k-2}=-1,a_{4k}=1\, (k=1,2,\cdots,m)$时,有$a_1=0,S(A_n)=0$;

当$n=4m+2$或$n=4m+3$ ($m\in\mathbb{N}$)时, $n(n-1)$不能被$4$整除,此时不存在$E$数列$A_n$,使得$a_1=0,S(A_n)=0$.
\end{enumerate}
\end{Proof}

 

\begin{Example}
(2012北京)设$A$是由$m\times n$个实数组成的$m$行$n$列的数表,满足:每个数的绝对值不大于$1$,且所有数的和为零.记$S(m,n)$为所有这样的数表构成的集合.

对于$A=S(m,n)$,记$r_i(A)$为$A$的第$i$行各数之和$(1\leq i\leq m)$, $c_i(A)$为$A$的第$j$列各数之和$(1\leq j\leq n)$.

记$k(A)$为$|r_1(A)|,|r_2(A)|,\cdots,|r_m(A)|,|c_1(A)|,|c_2(A)|,\cdots,|c_n(A)|$中的最小值.
\begin{enumerate}
\item[(I)] 对如下数表$A$,求$k(A)$的值;
\begin{figure}[!htbp]
\centering
\begin{tabular}{|c|c|c|}
\hline
% after \\: \hline or \cline{col1-col2} \cline{col3-col4} ...
$1$ & $1$ & $-0.8$ \\ \hline
$0.1$ & $-0.3$ & $-1$ \\
\hline
\end{tabular}
\end{figure}

\item[(II)] 设数表$A\in S(2,3)$形如
\begin{figure}[!htbp]
\centering
\begin{tabular}{|c|c|c|}
\hline
% after \\: \hline or \cline{col1-col2} \cline{col3-col4} ...
$1$ & $1$ & $c$ \\ \hline
$a$ & $b$ & $-1$ \\
\hline
\end{tabular}
\end{figure}
求$k(A)$的最大值;

\item[(III)] 给定正整数$t$,对于所有的$A\in S(2,2t+1)$,求$k(A)$的最大值.
\end{enumerate}
\end{Example}
\begin{Proof}
\begin{enumerate}
\item[(I)] 由题意可知$r_1(A)=1.2,r_2(A)=-1.2,c_1(A)=1.1,c_2(A)=0.7,c_3(A)=-1.8$,所以$k(A)=0.7$.

\item[(II)] 先用反证法证明$k(A)\leq 1$.

若$k(A)>1$,则$|c_1(A)|=|a+1|=a+1>1$,所以$a>0$.同理可知$b>0$,所以$a+b>0$.由题目所有数和为$0$,即$a+b+c=-1$,所以$c=-1-a-b<-1$,与题目条件矛盾,所以$k(A)\leq 1$.

易知当$a=b=0$时, $k(A)=1$存在.所以$k(A)$的最大值为$1$.

\textbf{注:}事实上,只需求$\max\{\min\{a+1,b+1,|1-a-b|\}\}$.


\item[(III)] $k(A)$的最大值为$\displaystyle\frac{2t+1}{t+2}$.

首先构造满足$\displaystyle k(A)=\frac{2t+1}{t+2}$的$A=\{a_{i,j}\}(i=1,2;j=1,2,\cdots,2t+1)$:
\begin{align*}
a_{1,1}&=a_{1,2}=\cdots=a_{1,t}=1,\\
a_{1,t+1}&=a_{1,t+2}=\cdots=a_{1,2t+1}=-\frac{t-1}{t+2},\\
a_{2,1}&=a_{2,2}=\cdots=a_{2,t}=\frac{t^2+t+1}{t(t+2)},\\
a_{2,t+1}&=a_{2,t+2}=\cdots=a_{2,2t+1}=-1.
\end{align*}
经计算知, $A$中每个元素的绝对值都小于$1$,所有元素之和为$0$,且
\begin{align*}
|r_1(A)|&=|r_2(A)|=\frac{2t+1}{t+2},\\
|c_1(A)|&=|c_2(A)|=\cdots=|c_t(A)|=1+\frac{t^2+t+1}{t(t+2)}
>1+\frac{t+1}{t+2}>\frac{2t+1}{t+2},\\
|c_{t+1}(A)|&=|c_{t+2}(A)|=\cdots=|c_{2t+1}(A)|=1+\frac{t-1}{t+2}=\frac{2t+1}{t+2}.
\end{align*}

下面证明$\displaystyle\frac{2t+1}{t+2}$是最大值.若不然,则存在一个数表$A\in S(2,2t+1)$,使得$\displaystyle k(A)=x>\frac{2t+1}{t+2}$.

由$k(A)$的定义知$A$的每一列两个数之和的绝对值都不小于$x$,而两个绝对值不超过$1$的数的和,其绝对值不超过$2$,故$A$的每一列两个数之和的绝对值都在区间$[x,2]$中.由于$x>1$,故$A$的每一列两个数符号均与列和的符号相同,且绝对值均不小于$x-1$.

设$A$中有$g$列的列和为正,有$h$列的列和为负,由对称性不妨设$g<h$,则$g\leq t,h\geq t+1$.另外,由对称性不妨设$A$的第一行行和为正,第二行行和为负.

考虑$A$的第一行,由前面结论知$A$的第一行有不超过$t$个正数和不少于$t+1$个负数,每个正数的绝对值不超过$1$ (即每个正数均不超过$1$),每个负数的绝对值不小于$x-1$ (即每个负数均不超过$1-x$).因此
\begin{align*}
|r_1(A)|&=r_1(A)\leq t\cdot 1+(t+1)(1-x)\\
&=2t+1-(t+1)x=x+(2t+1-(t+2)x)<x,
\end{align*}
故$A$的第一行行和的绝对值小于$x$,与假设矛盾.因此$k(A)$的最大值为$\displaystyle\frac{2t+1}{t+2}$.
\end{enumerate}
\end{Proof}

\begin{Example}
(2013北京)已知$\{a_n\}$是由非负整数组成的无穷数列,该数列前$n$项的最大值记为$A_n$,第$n$项之后各项$a_{n+1},a_{n+2},\dots$的最小值记为$B_n$, $d_n=A_n-B_n$.
\begin{enumerate}
\item[(I)] 若$\{a_n\}$为$2,1,4,3,2,1,4,3\cdots$,是一个周期为$4$的数列(即对任意$n\in\mathbb{N}^\ast$, $a_{n+4}=a_n$),写出$d_1,d_2,d_3,d_4$的值;

\item[(II)] 设$d$是非负整数,证明: $d_n=-d\, (n=1,2,3,\cdots)$的充分必要条件为$\{a_n\}$是公差为$d$的等差数列;

\item[(III)] 证明:若$a_1=2,d_n=1\, (n=1,2,3,\cdots)$, 则$\{a_n\}$的项只能是$1$或者$2$,且有无穷多项为$1$.
\end{enumerate}
\end{Example}
\begin{Proof}
\begin{enumerate}
\item[(I)] $d_1=d_2=1,d_3=d_4=3$.

\item[(II)] (充分性)因为$\{a_n\}$是公差为$d$的等差数列,且$D\geq 0$,所以$a_1\leq a_2\leq\cdots\leq a_n\leq\cdots$.因此$A_n=a_n,B_n=a_{n+1},d_n=a_n-a_{n+1}=-d\, (n=1,2,3,\cdots)$.

(必要性)因为$d_n=-d\leq 0\, (n=1,2,3,\cdots)$,所以$A_n=B_n+d_n\leq B_n$.又因为$a_n\leq A_n,a_{n+1}\geq B_n$,所以$a_n\leq a_{n+1}$.于是, $A_n=a_n,B_n=a_{n+1}$,因此$a_{n+1}-a_n=B_n-A_n=-d_n=d$,即$\{a_n\}$是公差为$d$的等差数列.

\item[(III)] 因为$a_1=2,d_1=1$,所以$A_1=a_1=2,B_1=A_1-d_1=1$.故对任意$n\geq 1$, $a_n\geq B_1=1$.

假设$\{a_n\}(n\geq 2)$中存在大于$2$的项.设$m$为满足$a_m>2$的最小正整数,则$m\geq 2$,并且对任意$1\leq k<m$, $a_k\leq 2$.又因为$a_1=2$,所以$A_{m-1}=2$,且$A_m=a_m>2$.于是, $B_m=A_m-d_m>2-1=1,B_{m-1}=\min\{a_m,B_m\}\geq 2$.故$d_{m-1}=A_{m-1}-B_{m-1}\leq 2-2=0$,与$d_{m-1}=1$矛盾.所以对于任意$n\geq 1$,有$a_n\leq 2$,即非负整数列$\{a_n\}$的各项只能为$1$或$2$.

因为对任意$n\geq 1$, $a_n\leq 2=a_1$,所以$A_n=2$.故$B_n=A_n-d_n=2-1=1$.

因此对于任意正整数$n$,存在$m$满足$m>n$,且$a_m=1$,即数列$\{a_n\}$有无穷多项为$1$.
\end{enumerate}
\end{Proof}


\begin{Example}
(2012北京)设$A$是由$m\times n$个实数组成的$m$行$n$列的数表,满足:每个数的绝对值不大于$1$,且所有数的和为零.记$S(m,n)$为所有这样的数表构成的集合.

对于$A=S(m,n)$,记$r_i(A)$为$A$的第$i$行各数之和$(1\leq i\leq m)$, $c_i(A)$为$A$的第$j$列各数之和$(1\leq j\leq n)$.

记$k(A)$为$|r_1(A)|,|r_2(A)|,\cdots,|r_m(A)|,|c_1(A)|,|c_2(A)|,\cdots,|c_n(A)|$中的最小值.
\begin{enumerate}
\item[(I)] 对如下数表$A$,求$k(A)$的值;
\begin{figure}[!htbp]
\centering
\begin{tabular}{|c|c|c|}
\hline
% after \\: \hline or \cline{col1-col2} \cline{col3-col4} ...
$1$ & $1$ & $-0.8$ \\ \hline
$0.1$ & $-0.3$ & $-1$ \\
\hline
\end{tabular}
\end{figure}

\item[(II)] 设数表$A\in S(2,3)$形如
\begin{figure}[!htbp]
\centering
\begin{tabular}{|c|c|c|}
\hline
% after \\: \hline or \cline{col1-col2} \cline{col3-col4} ...
$1$ & $1$ & $c$ \\ \hline
$a$ & $b$ & $-1$ \\
\hline
\end{tabular}
\end{figure}
求$k(A)$的最大值;

\item[(III)] 给定正整数$t$,对于所有的$A\in S(2,2t+1)$,求$k(A)$的最大值.
\end{enumerate}
\end{Example}
\begin{Proof}

\end{Proof}

计算行列式
\[\left| \begin{array} { c c c c } { a } & { b } & { c } & { d } \\ { - b } & { a } & { - d } & { c } \\ { - c } & { d } & { a } & { - b } \\ { - d } & { - c } & { b } & { a } \end{array} \right|.\]

心形线:
\[r=a(1-\sin\theta)\]

\[x ^ { 2 } + \left( y - \sqrt [ 3 ] { x ^ { 2 } } \right) ^ { 2 } = 1\]

\[5 x ^ { 2 } - 6 | x | y + 5 y ^ { 2 } = 128\]

\[\left( x ^ { 2 } + y ^ { 2 } - 1 \right) ^ { 3 } - x ^ { 2 } y ^ { 3 } = 0\]

\[\left( x ^ { 2 } + \frac { 9 } { 4 } y ^ { 2 } + z ^ { 2 } - 1 \right) ^ { 3 } - x ^ { 2 } z ^ { 3 } - \frac { 9 } { 80 } y ^ { 2 } z ^ { 3 } = 0,\quad -3\leq x,y,z\leq 3\]

\[\left( 2 x ^ { 2 } + y ^ { 2 } + z ^ { 2 } - 1 \right) ^ { 2 } - \frac { x ^ { 2 } z ^ { 2 } } { 10 } - y ^ { 2 } z ^ { 2 } = 0,\quad -3\leq x,y,z\leq 3 \]

\begin{Example}
(2013北京)已知$\{a_n\}$是由非负整数组成的无穷数列,该数列前$n$项的最大值记为$A_n$,第$n$项之后各项$a_{n+1},a_{n+2},\dots$的最小值记为$B_n$, $d_n=A_n-B_n$.
\begin{enumerate}
\item[(I)] 若$\{a_n\}$为$2,1,4,3,2,1,4,3\cdots$,是一个周期为$4$的数列(即对任意$n\in\mathbb{N}^\ast$, $a_{n+4}=a_n$),写出$d_1,d_2,d_3,d_4$的值;

\item[(II)] 设$d$是非负整数,证明: $d_n=-d\, (n=1,2,3,\cdots)$的充分必要条件为$\{a_n\}$是公差为$d$的等差数列;

\item[(III)] 证明:若$a_1=2,d_n=1\, (n=1,2,3,\cdots)$, 则$\{a_n\}$的项只能是$1$或者$2$,且有无穷多项为$1$.
\end{enumerate}
\end{Example}
\begin{Proof}
\begin{enumerate}
\item[(I)] $d_1=d_2=1,d_3=d_4=3$.

\item[(II)] (充分性)因为$\{a_n\}$是公差为$d$的等差数列,且$D\geq 0$,所以$a_1\leq a_2\leq\cdots\leq a_n\leq\cdots$.因此$A_n=a_n,B_n=a_{n+1},d_n=a_n-a_{n+1}=-d\, (n=1,2,3,\cdots)$.

(必要性)因为$d_n=-d\leq 0\, (n=1,2,3,\cdots)$,所以$A_n=B_n+d_n\leq B_n$.又因为$a_n\leq A_n,a_{n+1}\geq B_n$,所以$a_n\leq a_{n+1}$.于是, $A_n=a_n,B_n=a_{n+1}$,因此$a_{n+1}-a_n=B_n-A_n=-d_n=d$,即$\{a_n\}$是公差为$d$的等差数列.

\item[(III)] 因为$a_1=2,d_1=1$,所以$A_1=a_1=2,B_1=A_1-d_1=1$.故对任意$n\geq 1$, $a_n\geq B_1=1$.

假设$\{a_n\}(n\geq 2)$中存在大于$2$的项.设$m$为满足$a_m>2$的最小正整数,则$m\geq 2$,并且对任意$1\leq k<m$, $a_k\leq 2$.又因为$a_1=2$,所以$A_{m-1}=2$,且$A_m=a_m>2$.于是, $B_m=A_m-d_m>2-1=1,B_{m-1}=\min\{a_m,B_m\}\geq 2$.故$d_{m-1}=A_{m-1}-B_{m-1}\leq 2-2=0$,与$d_{m-1}=1$矛盾.所以对于任意$n\geq 1$,有$a_n\leq 2$,即非负整数列$\{a_n\}$的各项只能为$1$或$2$.

因为对任意$n\geq 1$, $a_n\leq 2=a_1$,所以$A_n=2$.故$B_n=A_n-d_n=2-1=1$.

因此对于任意正整数$n$,存在$m$满足$m>n$,且$a_m=1$,即数列$\{a_n\}$有无穷多项为$1$.
\end{enumerate}
\end{Proof}

\begin{Example}
(2014北京)对于数对序列$P:(a_1,b_1),(a_2,b_2),\cdots,(a_n,b_n)$,记$T_1(P)=a_1+b_1$, $T_k(P)=b_k+\max\{T_{k-1}(P),a_1+a_2+\cdots+a_k\}(2\leq k\leq n)$,其中$\max\{T_{k-1}(P),a_1+a_2+\cdots+a_k\}$表示$T_{k-1}(P)$和$a_1+a_2+\cdots+a_k$两个数中最大的数.
\begin{enumerate}
\item[(I)] 对于数对序列$P(2,5),P(4,1)$,求$T_1(P),T_2(P)$的值;

\item[(II)] 记$m$为$a,b,c,d$四个数中最小值,对于由两个数对$(a,b),(c,d)$组成的数对序列$P(a,b),(c,d)$和$P'(c,d),(a,b)$,试分别对$m=a$和$m=d$的两种情况比较$T_2(P)$和$T_2(P')$的大小;

\item[(III)] 在由$5$个数对$(11,8),(5,2),(16,11),(11,11),(4,6)$组成的所有数对序列中,写出一个数对序列$P$使$T_5(P)$最小,并写出$T_5(P)$的值. (只需写出结论).
\end{enumerate}
\end{Example}
\begin{Proof}
\begin{enumerate}
\item[(I)] $T_1(P)=2+5=7,T_2(P)=1+\max\{T_1(P),2+4\}=1+\max\{7,6\}=8$.

\item[(II)] 当$m=a$时, $T_1(P)=a+b,T_2(P)=d+\max\{a+b,a+c\}=a+d+\max\{b,c\}$;

$T_1(P')=c+d,T_2(P')=b+\max\{c+d,c+a\}=b+c+\max\{a,d\}=b+c+d$;

因为$a$是$a,b,c,d$中最小的数,所以$a+\max\{b,c\}\leq b+c$,从而$T_2(P)\leq T_2(P')$;

当$m=d$时, $T_1(P)=a+b,T_2(P)=d+\max\{a+b,a+c\}=a+d+\max\{b,c\}$;

$T_1(P')=c+d,T_2(P')=b+\max\{c+d,c+a\}=b+c+\max\{a,d\}=a+b+c$;

因为$d$是$a,b,c,d$中最小的数,所以$d+\max\{b,c\}\leq b+c$,从而$T_2(P)\leq T_2(P')$.

综上,这两种情况下都有$T_2(P)\leq T_2(P')$.

\item[(III)] 数对序列$P: (4,6),(11,11),(16,11),(11,8),(5,2)$的$T_5(P)$值最小;
$T_1(P)=10,T_2(P)=26,T_3(P)=42,T_4(P)=50,T_5(P)=52$.
\end{enumerate}
\end{Proof}


\begin{Example}
(2015北京)已知数列$\{a_n\}$满足: $a_1\in\mathbb{N}^\ast,a_1\leq 36$,且
\[a_{n+1}=\begin{cases}
2a_n, & a_n\leq 18 \\
2a_n-36, & a_n>18
\end{cases}(n=1,2,\cdots)\]
记集合$M=\{a_n|n\in N^\ast\}$.
\begin{enumerate}
\item[(I)] 若$a_1=6$,写出集合$M$的所有元素;

\item[(II)] 如集合$M$存在一个元素是$3$的倍数,证明: $M$的所有元素都是$3$的倍数;

\item[(III)] 求集合$M$的元素个数的最大值.
\end{enumerate}
\end{Example}
\begin{Proof}
\begin{enumerate}
\item[(I)] $6,12,24$.

\item[(II)] 因为集合$M$存在一个元素是$3$的倍数,所以不妨设$a_k$是$3$的倍数.

由$a_{n+1}=\begin{cases}
2a_n, & a_n\leq 18 \\
2a_n-36, & a_n>18
\end{cases}$可归纳证明对任意$n\geq k$, $a_n$是$3$的倍数.

如果$k=1$, 则$M$的所有元素都是$3$的倍数.

如果$k>1$,因为$a_k=2a_{k-1}$,或$a_k=2a_{k-1}-36$,所以$2a_{k-1}$是$3$的倍数,于是$a_{k-1}$是$3$的倍数;类似可得, $a_{k-2},\cdots,a_1$都是$3$的倍数.从而对任意$n\geq 1$, $a_n$是$3$的倍数.因此$M$的所有元素都是$3$的倍数.

综上,若集合$M$存在一个元素是$3$的倍数,则$M$的所有元素都是$3$的倍数.

\item[(III)] 由$a_1\leq 36,a_{n}=\begin{cases}
2a_{n-1}, & a_{n-1}\leq 18 \\
2a_{n-1}-36, & a_{n-1}>18
\end{cases}$可归纳证明$a_n\leq 36\, (n=2,3,\cdots)$.因为$a_1$是正整数, $a_2=\begin{cases}
2a_1, & a_1\leq 18, \\
2a_1-36, & a_1>18,
\end{cases}$
所以$a_2$是$2$的倍数.


从而当$n\geq 3$时, $a_n$是$4$的倍数.

如果$a_1$是3的倍数,由(II)知对所有正整数$n$, $a_n$是$3$的倍数.

因此当$n\geq 3$时, $an=\{12,24,36\}$,这时$M$的元素个数不超过$5$.

如果$a_1$不是$3$的倍数,由(II)知对所有正整数$n$, $a_n$不是$3$的倍数.

因此当$n\geq 3$时, $a_n\in \{4,8,16,20,28,32\}$.这时$M$的元素个数不超过$8$.

当$a_1=1$时, $M=\{1,2,4,8,16,20,28,32\}$有8个元素.

综上可知,集合$M$的元素个数的最大值为$8$.
\end{enumerate}
\end{Proof}


\begin{Example}
(2016北京)设数列$A:a_1,a_2,\cdots,a_N\,(N\geq 2)$,如果对小于$n\, (2\leq n\leq N)$的每个正整数$k$都有$a_k<a_n$,则称$n$是数列$A$的一个“$G$时刻”,记$G(A)$是数列$A$的所有“$G$时刻”组成的集合.
\begin{enumerate}
\item[(I)] 对数列$A:-2,2,-1,1,3$,写出$G(A)$的所有元素;

\item[(II)] 证明:若数列$A$中存在$a_n$使得$a_n>a_1$,则$G(A)\neq\emptyset$;

\item[(III)] 证明:若数列$A$满足$a_n-a_{n-1}\leq 1\, (n=2,3,\cdots,N)$,则$G(A)$的元素个数不小于$a_N-a_1$.
\end{enumerate}
\end{Example}
\begin{Proof}
\begin{enumerate}
\item[(I)] $G(A)$的元素为$2$和$5$.

\item[(II)] 因为存在$a_n$使得$a_n>a_1$,所以$\{i\in\mathbb{N}^\ast|2\leq i\leq N,a_i>a_1\}\neq \emptyset$.

记$m=\min\{i\in\mathbb{N}^\ast|2\leq i\leq N,a_i>a_1\}$,则$m\geq 2$,且对任意正整数$k< m$, $a_k\leq a_1<a_m$.因此$m\in G(A)$.从而$G(A)\neq \emptyset$.

\item[(III)] 当$a_N\leq a_1$时,结论成立.

以下设$a_N>a_1$.由(II)知$G(A)\neq\emptyset$.

设$G(A)=\{n_1,n_2,\cdots,n_p\},n_1<n_2<\cdots<n_p$.记$n_0=1$,则$a_{n_0}<a_{n_1}<a_{n_2}<\cdots<a_{n_p}$.

对$i=0,1,\cdots,p$,记$G_i=\left\{k\in\mathbb{N}^\ast|n_i<k\leq N,a_k>a_{n_i}\right\}$.

如果$G_i\neq\emptyset$,取$m_i=\min G_i$,则对任何$1\leq k<m_i,a_k\leq a_{n_i}<a_{m_i}$.从而$m_i\in G(A)$且$m_i=n_{i+1}$.

又因为$n_p$是$G(A)$中的最大元素,所以$G_p=\emptyset$.从而对任意$n_p\leq k\leq N,a_k\leq a_{n_p}$,特别地, $a_N\leq a_{n_p}$.

对$i=0,1,\cdots,p-1$, $a_{n_{i+1}-1}\leq a_{n_i}$.因此$a_{n_{i+1}}=a_{n_{i+1}-1}+(a_ {n_{i+1}}-a_{n_{i+1}-1})\leq a_ {n_i}+1$.所以
\[a_N-a_1\leq a_{n_p}-a_1=\sum_{i=1}^{p}(a_{n_i}-a_{n_{i-1}})\leq p.\]
因此$G(A)$的元素个数$p$不小于$a_N-a_1$.


\textbf{证法二.}猿题库.
\end{enumerate}
\end{Proof}

 

\begin{Example}
(2017北京)设$\{a_n\}$和$\{b_n\}$是两个等差数列,记$c_n=\max\{b_1-a_1n,b_2-a_2n,\cdots,b_n-a_nn\}\, (n=1,2,3,\cdots)$,其中$\max\{x_1,x_2,\cdots,x_s\}$表示$x_1,x_2,\cdots,x_s$这$s$个数中最大的数.
\begin{enumerate}
\item[(I)] 若$a_n=n,b_n=2n-1$,求$c_1,c_2,c_3$的值,并证明$\{c_n\}$是等差数列;

\item[(II)] 证明:或者对任意正数$M$,存在正整数$m$, 当$n\geq m$时, $\displaystyle\frac{c_n}{n}>M$;或者存在正整数$m$,使得$c_m,c_{m+1},c_ {m+2},\cdots$是等差数列.
\end{enumerate}
\end{Example}
\begin{Proof}
\begin{enumerate}
\item[(I)]
\begin{align*}
c_1&=b_1-a_1=1-1=0,\\
c_2&=\max\{b_1-2a_1,b_2-2a_2\}=\max\{1-2\times 1,3-2\times 2\}=-1,\\
c_3&=\max\{b_1-3a_1,b_2-3a_2,b_3-3a_3\}=\max\{1-3\times 1,3-3\times 2,5-3\times 3\}=-2,
\end{align*}
当$n\geq 3$时,
\[(b_{k+1}-na_{k+1})-(b_k-na_k)=(b_{k+1}-b_k)-n(a_{k+1}-a_k)=2-n<0,\]
所以$b_k-na_k$关于$n\in\mathbb{N}^\ast$单调递减.所以$c_n=\max\{b_1-a_1n,b_2-a_2n,\cdots,b_n-a_nn\}=b_1-a_1n=1-n$.所以对任意$n\geq 1$, $c_n=1-n$,于是$c_{n+1}-c_n=-1$,所以$\{c_n\}$是等差数列.

\item[(II)] 设$\{a_n\}$和$\{b_n\}$的公差分别为$d_1,d_2$,则
\[b_k-na_k=b_1+(k-1)d_2-[a_1+(k-1)d_1]n=b_1-a_1n+(d_2-nd_1)(k-1),\]
所以
\[c_n=\begin{cases}
b_1-a_1n+(n-1)(d_2-nd_1), & \mbox{当}\, d_2>nd_1\, \mbox{时},\\
b_1-a_1n, & \mbox{当}\, d_2\leq nd_1\, \mbox{时}.
\end{cases}\]
\begin{enumerate}
\item 当$d_1>0$时,取正整数$m>\frac{d_2}{d_1}$,则当$n\geq m$时, $nd_1>d_2$,因此$c_n=b_1-a_1n$.此时, $c_m,c_{m+1},c_ {m+2},\cdots$是等差数列.

\item 当$d_1=0$时,对任意$n\geq 1$, $c_n=b_1-a_1n+(n-1)\max\{d_2,0\}=b_1-a_1+(n-1)(\max\{d_2,0\}-a_1)$.此时, $c_1,c_2,c_3,\cdots,c_n,\cdots$是等差数列.

\item 当$d_1<0$时,当$n>\frac{d_2}{d_1}$时,有$nd_1<d_2$,所以
\begin{align*}
\frac{c_n}{n}=\frac{b_1-a_1n+(n-1)(d_2-nd_1)}{n}
=n(-d_1)+d_1-a_1+d_2+\frac{b_1-d_2}{n}\\
\geq n(-d_1)+d_1-a_1+d_2-|b_1-d_2|.
\end{align*}
对任意正数$M$,取正整数\[
m>\max \left\{ \frac{M+\left| b_1-d_2 \right|+a_1-d_1-d_2}{-d_1},\frac{d_2}{d_1} \right\},
\]
故当$n\geq m$时, $\displaystyle\frac{c_n}{n}>M$.
\end{enumerate}
\end{enumerate}
\end{Proof}

\begin{Example}
(2018北京)设$n$为正整数,集合$A=\{\alpha|\alpha=(t_1,t_2,\cdots,t_n),t_k\in\{0,1\},k=1,2,\cdots,n\}$.对于集合$A$中的任意元素$\alpha= (x_1,x_2,\cdots,x_n)$和$\beta=(y_1,y_2,\cdots,y_n)$,记
\[M(\alpha,\beta)=\frac{1}{2}\left[(x_1+y_1-|x_1-y_1|)+(x_2+y_2-|x_2-y_2|)
+\cdots+(x_n+y_n-|x_n-y_n|)\right].\]
\begin{enumerate}
\item[(I)] 当$n=3$时,若$\alpha= (1,1,0)$和$\beta=(0,1,1)$,求$M(\alpha,\alpha)$
和$M(\alpha,\beta)$的值;

\item[(II)] 当$n=4$时,设$B$是$A$的子集,且满足:对于$B$中的任意元素$\alpha,\beta$,当$\alpha,\beta$相同时, $M(\alpha,\beta)$是奇数;当$\alpha,\beta$不同时, $M(\alpha,\beta)$是偶数.求集合$B$中元素个数的最大值;

\item[(III)] 给定不小于$2$的$n$,设$B$是$A$的子集,且满足:对于$B$中的任意两个不同的元素$\alpha,\beta$, $M(\alpha,\beta)=0$.写出一个集合$B$,使其元素个数最多,并说明理由.
\end{enumerate}
\end{Example}
\begin{Proof}
\begin{enumerate}
\item[(I)] 因为$\alpha=(1,1,0),\beta=(0,1,1)$,所以
\begin{align*}
M(\alpha,\alpha)&=\frac{1}{2}\left[(1+1-|1-1|)+(1+1-|1-1|)+(0+0-|0-0|)\right]=2,\\
M(\alpha,\beta)&=\frac{1}{2}\left[(1+0-|1-0|)+(1+1-|1-1|)
+(0+1-|0-1|)\right]=1.
\end{align*}

\item[(II)] 设$\alpha=(x_1,x_2,x_3,x_4)\in B$,则$M(\alpha,\alpha)=x_1+x_2+x_3+x_4$.由题意知$x_1,x_2,x_3,x_4\in\{0,1\}$,且$M(\alpha,\beta)$为奇数,所以$x_1,x_2,x_3,x_4$中$1$的个数为$1$或$3$.
所以$B\subset\{(1,0,0,0),(0,1,0,0),(0,0,1,0),(0,0,0,1),
(0,1,1,1),(1,0,1,1),(1,1,0,1),(1,1,1,0)\}$.将上述集合中的元素分成如下四组: $(1,0,0,0),(1,1,1,0);(0,1,0,0),(1,1,0,1);(0,0,1,0),(1,0,1,1);(0,0,0,1),
(0,1,1,1)$.经验证,对于每组中两个元素$\alpha,\beta$,均有$M(\alpha,\beta)=1$.

所以每组中的两个元素不可能同时是集合$B$的元素.所以集合$B$中元素的个数不超过$4$.

又集合$\{(1,0,0,0),(0,1,0,0),(0,0,1,0),(0,0,0,1)\}$满足条件,所以集合$B$中元素个数的最大值为$4$.

\item[(III)] 设$S_k=\{(x_1,x_2,\cdots,x_n)|(x_1,x_2,\cdots,x_n)\in A,x_k=1,x_1=x_2=\cdots=x_{k-1}=0\}\,(k=1,2,\cdots,n)$, $S_{n+1}=\{(x_1,x_2,\cdots,x_n)|x_1=x_2=\cdots=x_n=0\}$,则$A=S_1\cup S_2\cup\cdots S_{n+1}$.对于$S_k\, (k=1,2,\cdots,n-1)$中的不同元素$\alpha,\beta$,经验证, $M(\alpha,\beta)\geq 1$.所以$S_k\, (k=1,2,\cdots,n-1)$中的两个元素不可能同时是集合$B$的元素.所以$B$中元素的个数不超过$n+1$.

取$e_k=(x_1,x_2,\cdots,x_n)\in S_k$且$x_{k+1}=\cdots=x_n=0\,(k=1,2,\cdots,n-1)$.
令$B=(e_1,e_2,\cdots,e_{n-1})\cup S_n\cup S_{n+1}$,则集合$B$的元素个数为$n+1$,且满足条件.故$B$是一个满足条件且元素个数最多的集合.
\end{enumerate}
\end{Proof}

\begin{Example}
(2008人大附中10月理科月考)用$|X|$表示有限集$X$的元素个数,对由正整数组成的集合$A,B$,定义
\[A+B=\{x|x=a+b,a\in A,b\in B\}.\]
\begin{enumerate}
\item[(I)] 设集合$A=\{1,2,3,4,5,6,7,8\},B=\{4,8,16,32\}$,求$|A+B|$;

\item[(II)] 若$|A|=8,|B|=4$,求$|A+B|$的最小值;

\item[(III)] 若$|A|=8,|B|=4$,且$A$满足当$a,b,c,d\in A,a+b=c+d$时, $\{a,b\}=\{c,d\}$,求$|A+B|$的最小值.
\end{enumerate}
\end{Example}
\begin{Proof}

\end{Proof}


\begin{Example}
(2013湖北)设$n$是正整数, $r$为正有理数.
\begin{enumerate}
\item[(I)] 求函数$f(x)=(1+x)^{r+1}-(r+1)x-1 \,(x>-1)$的最小值;

\item[(II)] 证明:
\[\frac{n^{r+1}-(n-1)^{r+1}}{r+1}<n^r<\frac{(n+1)^{r+1}-n^{r+1}}{r+1};\]

\item[(III)] 设$x\in\mathbb{R}$,记$\lceil 2 \rceil =2,\lceil \pi \rceil =4,\lceil -\frac{3}{2} \rceil =-1$.令$S=\sqrt[3]{81}+\sqrt[3]{82}+\sqrt[3]{83}+\cdots+\sqrt[3]{125}$,求$\lceil S \rceil$的值.

(参考数据: $80^{4/3}\approx 344.7,81^{4/3}\approx 350.5,124^{4/3}\approx 618.3,126^{4/3}\approx 631.7$)
\end{enumerate}
\end{Example}
\begin{Proof}

\end{Proof}


(2014北京)对于数对序列$P:(a_1,b_1),(a_2,b_2),\cdots,(a_n,b_n)$,记$T_1(P)=a_1+b_1$, $T_k(P)=b_k+\max\{T_{k-1}(P),a_1+a_2+\cdots+a_k\}(2\leq k\leq n)$,其中$\max\{T_{k-1}(P),a_1+a_2+\cdots+a_k\}$表示$T_{k-1}(P)$和$a_1+a_2+\cdots+a_k$两个数中最大的数.
\begin{enumerate}
\item[(1)] 对于数对序列$P(2,5),P(4,1)$,求$T_1(P),T_2(P)$的值;

\item[(2)] 记$m$为$a,b,c,d$四个数中最小值,对于由两个数对$(a,b),(c,d)$组成的数对序列$P(a,b),(c,d)$和$P(a,c),(b,d)$,试分别对$m=a$和$m=d$的两种情况比较$T_2(P)$和$T_2(P')$的大小;

\item[(3)] 在由$5$个数对$(11,8),(5,2),(16,11),(11,11),(4,6)$组成的所有数对序列中,写出一个数对序列$P$使$T_5(P)$最小,并写出$T_5(P)$的值. (只需写出结论).
\end{enumerate}

\begin{Example}
(2008北京)
\end{Example}
\begin{Proof}

\end{Proof}


\begin{Example}
(2008北京)
\end{Example}
\begin{Proof}

\end{Proof}


\begin{Example}
(2008北京)
\end{Example}
\begin{Proof}

\end{Proof}

 

 


\begin{Example}
(2008人大附中10月理科月考)用$|X|$表示有限集$X$的元素个数,对由正整数组成的集合$A,B$,定义
\[A+B=\{x|x=a+b,a\in A,b\in B\}.\]
\begin{enumerate}
\item[(I)] 设集合$A=\{1,2,3,4,5,6,7,8\},B=\{4,8,16,32\}$,求$|A+B|$;

\item[(II)] 若$|A|=8,|B|=4$,求$|A+B|$的最小值;

\item[(III)] 若$|A|=8,|B|=4$,且$A$满足当$a,b,c,d\in A,a+b=c+d$时, $\{a,b\}=\{c,d\}$,求$|A+B|$的最小值.
\end{enumerate}
\end{Example}
\begin{Proof}

\end{Proof}


\begin{Example}
已知抛物线$\displaystyle C:y=\frac{1}{2}x^2$与直线$l:y=kx-1$没有公共点,设点$P$为直线$l$上的动点,过$P$作抛物线$C$的两条切线, $A,B$为切点.
\begin{enumerate}
\item[(I)] 证明:直线$AB$恒过定点$Q$;

\item[(II)] 若点$P$与(I)中的定点$Q$的连线交抛物线$C$于$M,N$两点,证明:
\[\frac{|PM|}{|PN|}=\frac{|QM|}{|QN|}.\]
\end{enumerate}
\end{Example}
\begin{Proof}

\end{Proof}


\begin{Example}
过点$P(a,-2)$作抛物线$C:x^2=4y$的两条切线,切点分别为$A(x_1,y_1),B(x_2,y_2)$.
\begin{enumerate}
\item[(I)] 证明: $x_1x_2+y_1y_2$为定值;

\item[(II)] 记$\triangle PAB$的外接圆的圆心为点$M$,点$F$是抛物线$C$的焦点, 对任意实数$a$,试判断以$PM$为直径的圆是否恒过点$F$?并说明理由.
\end{enumerate}
\end{Example}
%http://www.1010jiajiao.com/gzsx/shiti_id_d78bda373ee11dbb6218e37b79ec48a5
\begin{Proof}

\end{Proof}

posted on 2019-01-14 23:54  Eufisky  阅读(232)  评论(0编辑  收藏  举报

导航